LSAT and Law School Admissions Forum

Get expert LSAT preparation and law school admissions advice from PowerScore Test Preparation.

User avatar
 Dave Killoran
PowerScore Staff
  • PowerScore Staff
  • Posts: 5853
  • Joined: Mar 25, 2011
|
#27320
Complete Question Explanation

The correct answer choice is (D)

Answer choice (A) is incorrect because if the main-dish recipe has three flavorings, then the appetizer recipe will have four flavorings, and from the first rule the appetizer recipe can only have at most three flavorings.

Answer choice (B) is incorrect because F and N cannot be included in the same recipe per the second rule.

Answer choice (C) is incorrect because from the third rule one of S or T must be included in the main-dish recipe.

Answer choice (D) is the correct answer choice.

Answer choice (E) is incorrect because S and T cannot be included in the same recipe.
 kferry1126@yahoo.com
  • Posts: 1
  • Joined: Mar 26, 2021
|
#87873
Where does it say that in the rules? The third rule in the workbook states "Saffron is not included in the same recipe as turmeric.".
User avatar
 Ryan Twomey
PowerScore Staff
  • PowerScore Staff
  • Posts: 141
  • Joined: Mar 04, 2021
|
#87931
Hey Kferry,

So because this is a binary game, when the third rule states, S and T cannot be included in the same dish, we get that at least one of them has to be in the appetizer and one of them has to be in the main. So we cannot have a complete list of main dishes with neither of them present, because that would be forcing both of them into the appetizer category.

In your main diagram, you should have placeholders S/T and F/N both present in both the appetizer and the maindish category. There are quite a lot of tricks in these binary two-group games, but the not-block trick is probably the most important.

I hope this makes sense, and I wish you all of the luck in your studies.

Best,
Ryan

Get the most out of your LSAT Prep Plus subscription.

Analyze and track your performance with our Testing and Analytics Package.